diff --git a/inputs/lecture_17.tex b/inputs/lecture_17.tex new file mode 100644 index 0000000..94551b3 --- /dev/null +++ b/inputs/lecture_17.tex @@ -0,0 +1,240 @@ +\lecture{17}{2023-12-18}{Large cardinals} + + +\begin{definition} + \begin{itemize} + \item A cardinal $\kappa$ is called \vocab{weakly inaccessible} + iff $\kappa$ is uncountable,\footnote{dropping this we would get that $\aleph_0$ is inaccessible} + regular and $\forall \lambda < \kappa.~\lambda^+ < \kappa$. + \item A cardinal $\kappa$ is \vocab[inaccessible!strongly]{(strongly) inaccessible} + iff $\kappa$ is uncountable, regular + and $\forall \lambda < \kappa.~2^{\lambda} < \kappa$. + \end{itemize} +\end{definition} +\begin{remark} + Since $2^{\lambda} \ge \lambda^+$, + strongly inaccessible cardinals are weakly inaccessible. + + If $\GCH$ holds, the notions coincide. +\end{remark} + +\begin{theorem} + If $\kappa$ is inaccessible, + then $V_\kappa \models \ZFC$.\footnote{More formally $(V_{\kappa}, \in ) \models\ZFC$.} +\end{theorem} +\begin{proof} + Since $\kappa$ is regular, \AxRep works. + Since $2^{\lambda} < \kappa$, + \AxPow works. + The other axioms are trivial. + \todo{Exercise} +\end{proof} +\begin{corollary} + $\ZFC$ does not prove the existence of inaccessible + cardinals, unless $\ZFC$ is inconsistent. +\end{corollary} +\begin{proof} + If $\ZFC$ is consistent, + it can not prove that it is consistent. + In particular, it can not prove the existence of a model of $\ZFC$. +\end{proof} + +\begin{definition}[Ulam] + A cardinal $\kappa > \aleph_0$ is \vocab{measurable} + iff there is an ultrafilter $U$ on $\kappa$, + such that $U$ is not principal\footnote{% + i.e.~$\{\xi\} \not\in U$ for all $\xi < \kappa$% + } + and + if $\theta < \kappa$ + and $\{X_i : i < \theta\} \subseteq U$, + then $\bigcap_{i < \theta} X_i \in U$ +\end{definition} + +\begin{goal} + We want to prove + that if $\kappa$ is measurable, + then $\kappa$ is inaccessible + and there are $\kappa$ many + inaccessible cardinals below $\kappa$ + (i.e.~$\kappa$ is the $\kappa$\textsuperscript{th} inaccessible). +\end{goal} +\begin{theorem} + The following are equivalent: + \begin{enumerate} + \item $\kappa$ is a measurable cardinal. + \item There is an elementary embedding% + \footnote{Recall: $j\colon V \to M$ + is an \vocab{elementary embedding} iff $j''V = \{j(x) : x \in V\} \prec M$, + i.e.~for all formulae $\phi$ and $x_1,\ldots,x_k \in V$, + $V \models\phi(x_1,\ldots,x_u) \iff M \models\phi(j(x_1),\ldots,j(x_u))$.% + } + $j\colon V \to M$ with $M$ + transitive + such that $j\defon{\kappa} = \id$, + $j(\kappa) \neq \kappa$. + \end{enumerate} +\end{theorem} +\begin{proof} + 2. $\implies$ 1.: + Fox $j\colon V \to M$. + Let $U = \{X \subseteq \kappa : \kappa \in j(X)\}$. + We need to show that $U$ is an ultrafilter: + \begin{itemize} + \item Let $X,Y \in U$. + Then $\kappa \in j(X) \cap j(Y)$. + We have $M \models j(X \cap Y) = j(X) \cap j(Y)$, + and thus $j(X \cap Y) = j(X) \cap j(Y)$. + It follows that $X \cap Y \in U$. + \item Let $X\in U$ + and $X \subseteq Y \subseteq \kappa$. + Then $ \kappa \in j(X) \subseteq j(Y)$ + by the same argument, + so $Y \in U$. + \item We have $j(\emptyset) = \emptyset$ + (again $M \models j(\emptyset)$ is empty), + hence $\emptyset\not\in U$. + \item $\kappa \in U$ follows from $\kappa \in j(\kappa)$. + This is shown as follows: + \begin{claim} + For every ordinal $\alpha$, + $j(\alpha)$ is an ordinal such that $j(\alpha) \ge \alpha$. + \end{claim} + \begin{subproof} + $\alpha \in \Ord$ + can be written as + \[\forall x \in \alpha .~\forall y \in x.~y \in \alpha + \land \forall x \in \alpha .~ \forall y \in \alpha.~(x \in y \lor x = y \lor y \in x). + \] + So if $\alpha$ is an ordinal, + then $M \models \text{``$j(\alpha)$ is an ordinal''}$ + in the sense above. + Therefore $j(\alpha)$ really is an ordinal. + + If the claim fails, + we can pick the least $\alpha$ such that $j(\alpha) < \alpha$. + Then $M \models j(j(\alpha)) < j(\alpha)$, + i.e. $j(j(\alpha)) < j(\alpha)$ + contradicting the minimality of $\alpha$. + \end{subproof} + + Therefore as $j(\kappa) \neq \kappa$, + we have $j(\kappa) > \kappa$, + i.e.~$\kappa \in j(\kappa)$. + \item $U$ is an ultrafilter: + Let $X \subseteq \kappa$. + Then $\kappa \in j(\kappa) = j(X \cup (\kappa \setminus X)) = j(X) \cup j(\kappa \setminus X)$. + So $X \in U$ or $\kappa \setminus X \in U$. + + Let $\theta < \kappa$ + and $\{X_i : i < \theta\} \subseteq U$. + Then $\kappa \in j(X_i)$ for all $i < \theta$, + hence + \[ + \kappa \in \bigcap_{i < \theta} j(X_i) + = j\left( \bigcap_{i < \theta} X_i \right) \in U. + \] + This holds since $j(\theta) = \theta$ (as $\theta < \kappa$), + so $j(\langle X_i : i < \theta \rangle) = \langle j(X_i) : i < \theta \rangle$. + + Also if $\xi < \kappa$, + then $j(\{\xi\}) = \{\xi\}$ + so $\kappa \not\in j(\{\xi\})$ + and $\{\xi\} \not\in U$. + \end{itemize} + + + 1. $\implies$ 2. + Fix $U$. + Let ${}^{\kappa}V$ be the class of all function from $\kappa$ to $V$. + For $f,g \in {}^{\kappa}V$ define + $f \sim g :\iff \{\xi < \kappa : f(\xi) = g(\xi)\} \in U$. + This is an equivalence relation since $U$ is a filter. + Write $[f] = \{g \in {}^\kappa V : g \sim ~ f \land g \in V_\alpha \text{ for the least $\alpha$ such that there is some $h \in V_\alpha$ with $h \sim f$}\}$.% + \footnote{This is know as \vocab{Scott's Trick}. + Note that by defining equivalence classes + in the usual way (i.e.~without this trick), + one ends up with proper classes: + For $f\colon \kappa \to V$, + we can for example change $f(0)$ to be an arbitrary $V_{\alpha}$ + and get another element of $[f]$. + } + For any two such equivalence classes $[f], [g]$ + define + \[[f] \tilde{\in} [g] :\iff \{\xi < \kappa : f(\xi) \in g(\xi)\} \in U.\] + + This is independent of the choice of the representatives, + so it is well-defined. + Now write $\cF = \{[f] : f \in {}^{\kappa}V\}$ + and look at $(\cF, \tilde{\in})$. + + The key to the construction is \yaref{thm:los} (see below). + Given \yaref{thm:los}, + we may define an elementary embedding + $\overline{j}\colon (V, \in ) \to (\cF, \tilde{\in })$ + as follows: + +Let $\overline{j}(x) = [c_x]$, +where $c_x \colon \kappa \to \{x\}$ is the constant function +with value $x$. + +Then +\begin{IEEEeqnarray*}{rCl} + (V, \in ) \models \phi(x_1,\ldots,x_k)&\iff& + \{\xi < \kappa: (V, \in ) \models \phi(c_{x_1}(\alpha), \ldots, c_{x_k}(\alpha))\} \in U\\ + &\overset{\yaref{thm:los}}{\iff}& + (\cF, \tilde{\in }) \models \phi(\overline{j}(x_1), \ldots, \overline{j}(x_k)). +\end{IEEEeqnarray*} + +Let us show that $(\cF, \tilde{\in })$ +is well-founded. +Otherwise there is $\langle f_n : n < \omega \rangle$ +such that $f_n \in {}^\kappa V$ +and $[f_{n+1}] \tilde{\in } [f_n]$ for all $n < \omega$. + +Then $X_n \coloneqq \{\xi < \kappa : f_{n+1}(\xi) \in f_n(\xi)\} \in U$, +so $\bigcap X_n \in U$. +Let $\xi_0 \in \bigcap X_n$. +Then $f_0(\xi_0) \ni f_1(\xi_0) \ni f_2(\xi_0) \ni \ldots$ $\lightning$. + +Note that $\tilde{\in }$ is set-like, +therefore by the \yaref{lem:mostowski} +there is some transitive $M$ with $(\cF, \tilde{\in }) \overset{\sigma}{\cong} (M, \in )$. + +We can now define an elementary embedding $j\colon V \to M$ +by $j \coloneqq \sigma \circ \overline{j}$. + +It remains to show that $\alpha < \kappa \implies j(\alpha) = \alpha$. +This can be done by induction: +Fix $\alpha$. We already know $j(\alpha) \ge \alpha$. +Suppose $\beta \in j(\alpha)$. +Then $\beta = \sigma([f])$ for some $f$ +and $\sigma([f]) \in \sigma([c_{\alpha}])$, +i.e.~$[f] \tilde{\in} [c_\alpha]$. +Thus $\{\xi < \kappa : f(\xi) \in \underbrace{c_{\alpha}(\xi)}_{\alpha}\} \in U$. +Hence there is some $\delta < \alpha$ +such that +\[ + X_\delta \coloneqq \{\xi < \kappa : f(\xi) = \delta\} \in U, +\] +as otherwise $\forall \delta < \alpha. ~ \kappa \setminus X_\delta \in U$, +i.e.~$\emptyset = (\bigcap_{\delta < \alpha} \kappa \setminus X_{\delta}) \cap X \in U \lightning$. +We get $[f] = [c_{\delta}]$, +so $\beta = \sigma([f]) = \delta([c_{\delta}]) = j(\delta) = \delta$, +where for the last equality we have applied the induction hypothesis. +So $j(\alpha) \le \alpha$. + +% It is also easy to show $j(\kappa) > \kappa$. +\end{proof} + +\begin{theorem}[\L o\'s] + \yalabel{\L o\'s's Theorem}{\L o\'s}{thm:los} + For all formulae $\phi$ and for all $f_1, \ldots, f_k \in {}^{\kappa} V$, + \[ + (\cF, \tilde{\in}) \models \phi([f_1], \ldots, [f_k]) + \iff \{\xi < \kappa : (V, \in ) \models \phi(f_1(\xi), \ldots, f_k(\xi))\} \in U. + \] +\end{theorem} +\begin{proof} + Induction on the complexity of $\phi$. +\end{proof}